Poles and Polars – Another Useful Tool!

As the title suggests, this post is going to deal with the different aspects of poles and polars, which is a really great tool in case of proving problems using cyclic quadrilaterals and a lot of circular reasoning can be exploited by this. It is easier to deal with lines than with circles, so it is better to work with inversion, poles and polars etc – that’s why projective geometry is actually better in many aspects compared to plain euclidean geometry!

Anyways, for the time being let us know the definitions of poles and polars. Before continuing further, you have to note that a pole is a point, and a polar is a line (which is the opposite of Kedlaya, but yeah, notations don’t matter much. :) ) In any case, thanks to Rijul Saini for giving me a hand on this post, which we delayed for more than 8 months. At last, this is getting published on the blog, so Cheers, everyone! :)

Pole of a line
Take the circle \omega with respect to which you’re applying the polar map. Now, drop a perpendicular to the given line from the centre O of \omega to the given line. Name that point P. Now, invert P with respect to the circle. We get a point P' which lies on the same line as O,P. Then, P' is the Pole of the given line.

Polar of a point
Take the circle \omega with respect to which you’re applying the polar map. Let the centre of the circle be O. Now, invert the given point P with respect to the circle \omega and name the image as P'.
Now, Draw the line perpendicular to OP' which passes through the point P'. Then, this line is the Polar of the point P.

Now let us look into some theorems and lemmas that we will be using while solving problems.

Theorem 1:(La Hire’s theorem)
(i) Every point is the pole of its polar, and every line is the polar of its pole.
(ii) If P lies on the polar of Q then Q lies on the polar of P.

Proof:
(i) Again, direct from the definition.
(ii) \ P is a point on the polar of Q. First, extend OQ to the polar of Q and name that point Q'. If Q'= P then we are through. Otherwise, let the inverse of the point P wrt the circle be P'. Now, OQ \cdot OQ'=OP \cdot OP'=R^2. Now, by similar triangles, we have \angle OP'Q= \angle OQ'P=90^{\circ} and so we are through.\Box

Theorem 2:
Three points are collinear if and only if their polars are concurrent.
Proof:
I shall prove only the forward direction. The reverse direction is entirely analogous.
Let the three points be P,Q,R and their polars be p,q,r. Now, let the line through P,Q,R be l, and let the pole of l be L. Since, P,Q,R lie on the polar of L, therefore, L must lie on the polar of P,Q,R i.e. lines p,q,r. Thus, the lines p,q,r are concurrent at L.\Box

Now let us get to know the poles and polars of some points and lines that are considered to be important.

Polar of a point outside a circle with respect to itself.
Take an arbitrary point P outside a circle. Since the inverse of the point actually lies inside the circle, the polar will be a secant of the circle. Now, let us assume that the inverse point of P w.r.t. (O,r) is P'. The line perpendicular to \overline{OP'P} and passing through P' will intersect the circle in two points which will be symmetric w.r.t PP'. So let one of these two intersections be Q, and since PO\times P'O=r^2=PQ^2, we note that QOP'\sim POQ, so this leads to \angle OQP=90^{\circ}, ie:
Theorem 3:
The polar of a point P lying outside the circle is actually the chord of contact of the point P with respect to (O,r).

Polar of a point lying inside the circle does not have any special property that can be exploited alike to the previous one. However, we can take any arbitrary two chords passing through P, find their poles and join the two corresponding points to obtain the polar of that point P.

Now, let us move on to Theorem 4. The first name was coined by me, and the second one by Rijul Saini. Don’t assume that they are the original names of the theorem.
Theorem 4
(The fundamental theorem of poles and polars, or The theorem of two pascals)
Take a quadrilateral ABCD which is cyclic. If K=AC\cap BD then the polar of K with respect to \odot(ABCD) is the line joining AB\cap CD and AD\cap BC.
Proof 1.
The second name almost gives it away. This problem, however, was directly quoted and asked to prove in the Turkish TST 1993.
Anyways, using Pascal’s theorem in ACCBDD, we obtain AC\cap BD, CC\cap DD, BC\cap DA are collinear. Again using the same in CAADBB we obtain that CA\cap DB, AA\cap BB, AD\cap BC are collinear. Therefore it is noted that the points AC\cap BD, AD\cap BC, CC\cap DD, AA\cap BB are collinear. Denote this line as l.
Since l passes through CC\cap DD and AA\cap BB, which is the polar of AC\cap BD=K, we are done.\Box

Proof 2.
This uses harmonic divisions. :lol:
Let us take S=AB\cap CD, F=AD\cap BC, E=AC\cap BD. If the tangent to \odot(ABCD) at S touches it at M,N respectively such that M\in\overhat{DC}, denote X=MN\cap AD and Y=SE\cap BC, Z=MN\cap CD, T=SE\cap AD.
It can be shown easily that (S,X,A,B); (S,Z,C,D) and (S,E,Y,T) are harmonic. Then using (SXAB)=(SZCD) we note that AD,BC,ZX concur; and F,X,Z are collinear. Similarly we obtain that F,X,E and F,Z,E are collinear. Since XY\equiv MN, therefore E,F,M,N are collinear.
So F lies on the polar XZ of S w.r.t \odot(ABCD). By symmetry S lies on the polar YT of F w.r.t \odot(ABCD). So the intersection of these two polars will be E, and the polar of this pole will be FS. We are done! \Box
Image
There are some useful results that are correlated to harmonic divisions and poles and polars, which I explain in the following few theorems.
Theorem 5 (a).
If P is the pole of a line AB w.r.t \omega then any line \ell through P is cut harmonically by P,AB, and \omega.
Proof.
Let O be the centre of \omega, and let OP\cap \overleftrightarrow{AB}=B, \ \ell\cap \overleftrightarrow{AB}=A, \ PA\cap \omega = F.
Let \omega' be the circle passing through P,A,B. Then its centre is the midpoint O' of BP. If \omega\cap \omega' = C,D; then P is the pole of AB. So OP\cdpt OQ=OC^2. Then \omega and \omega' are orthogonal circles, which means that O'D is tangent to \omega. Therefore we obtain O'E\cdot O'F=O'D^2=O'P^2, and since O' is the midpoint of PA, from Theorem 1 – Harmonic Divisions, we are done.
import graph; size(300); real lsf = 0.5; pen dp = linewidth(0.7) + fontsize(10); defaultpen(dp); pen ds = black; pen ttttff =...
\Box

Theorem 5 (b).
If two lines p,q are self-conjugate lines and if p\cap q = T, then p,q are harmonic with the two tangents drawn to the circle from T.
Proof.
Let the poles of p,q be P,Q. if the two tangents from T meet the circle in X,Y.
Since the polar of P passes through T, from La Hire’s theorem we see that the polar of T passes through P, and similarly, Q. Then X,Y,P,Q are collinear. Now, from Theorem 5(a) we are done.
import graph;size(10cm);draw(circle((0,0),3));dot((1.5,4.45));dot((1.5,1.9));dot((6,0));dot((1.5,2.6));dot((1.5,-2.6));draw((...
\Box
Theorem 6.
Finally, if four points A,B,C,D form a harmonic range then their polars a,b,c,d will create a harmonic pencil.
Proof.
Let X be the pole of \overleftrightarrow{ABCD}, and let Y=OX\cap \overleftrightarrow{ABCD}.
If we draw four perpendicular lines XA', XB', XC', XD' perpendicular to OA,OB,OC,OD, respectively, then we have an interesting result. The polar of X passes through A, so from La Hire’s, the polar of A passes through X, and therefore XA' is the polar of A with respect to our aforementioned circle. It is obvious that the pencils X(A',B',C',D') and O(A,B,C,D) are equiangular pencils, and since O(A,B,C,D) are harmonic, therefore X(A',B',C',D') are also harmonic.\Box

Wow, that was enough of theory for a day already! Let us quickly look into some applications of our Dual Pascal and try to find boring solutions to some problems!

\boxed{E1}. Let XY be the diameter of a circle with two points P,Q on its circumference such that P is closer to X than Q. If PX and QY intersect at S outside the circle, and if the tangents at P,Q to the circle meet at R; then show that RS\perp XY.
Solution
Let T=PQ\cap XY. We already know that(from Theorem 4), S lies on the polar of T. The polar of R is \overline{PQT}, we see that R lies on the polar of T. So SR is the polar of T, and therefore we are done. \Box
import graph;unitsize(0.9cm);draw(circle((0,0),3));dot((-2.236,2));dot((3,0));dot((-3,0));dot((1.3, 2.7));draw((3,0)--(-12.33...

\boxed{E2}. Let ABCD be a tangential quadrilateral with incentre I. Let the opposite sides of ABCD meet each other at E,F; ie let E=AB\cap CD, F=AD\cap BC. Let its incircle touch the sides AB,BC,CD,DA at G,H,K,L, respectively. If P=GK\cap HL, then show that OP\perp EF.
Solution
Note that EG,EK are tangents to the incircle of ABCD, so that KG is the polar of E. Similarly the polar of F is HL. Therefore the pole of the line EF is KG\cap HL=P, and we are done. \Box

\boxed{E3}.(China 2006 Western MO) AB is the diameter of a circle with centre O. C is a point on AB, extended. A line through C cuts the circle with centre O at D,E. OF is a diameter of \odot(BOD) which has centre O_1. Let CF\cap \odot(BOD)=G. Prove that O,E,A,G lie on a circle.
Solution
Let P=AE\cap BD. So the polar of P with respect to the circle (O) passes through BA\cap DE=C and AD\cap BE=J. So PO\perp CJ, and it is obvious that G, O, P are collinear.
So by considering the power of P we have PD\cdot PB=PG\cdot PO=PE\cdot PA; and so E,A,O,G are concyclic points. \Box
Image

\boxed{E4}.(IMO 1985) A circle with center O passes through the vertices A and C of triangle ABC and intersects segments AB and BC again at distinct points K and N, respectively. The circumcircles of triangles ABC and KBN intersects at exactly two distinct points B and M. Prove that \angle OMB=90^{\circ}.
Solution
Note that the spiral similarity centered at M which sends A to C and K to N sends the midpoint of AK(say, M_1) to that of CN(say, M_2). So M is the center of unique spiral similarity that sends A to M_2
and C to M_1, and thus it follows that M,M_1,M_2,B are concyclic. Again since \angle OM_2B=\angle OM_1B, so O,M_1,M_2,B are concyclic, and OB is the diameter of the common circle. So, we are done. \Box

\boxed{E5}. Circles \omega_1 and \omega_2 meet at points O and M. Circle \omega, centered at O, meets circles \omega_1 and \omega_2 in four distinct points A,B,C and D, such that ABCD is a convex quadrilateral. Lines AB and CD meet at N_1. Lines AD and BC meet at N_2. Prove that N_1N_2\perp MO.
Solution
Actually this is equivalent to our last problem. Note that if \odot(ADN_1)\cap \odot(BCN_1)=K, then from simple angle chasing we can show that K=\odot(AOCK)\cap \odot(BODK). Also it is obvious that N_1N_2 passes through K\equiv M. All of this implies that N_1N_1\perp MO, as desired.
import graph; size(15cm); real labelscalefactor = 0.5; /* changes label-to-point distance */pen dps = linewidth(0.7) + fontsi...

\boxed{E6}. Let ABC be a triangle with incentre I. Reflections of I in BC,CA, AB are X,Y,Z. Prove that AX,BY,CZ are concurrent.
Note: This is a direct consequence of the isogonic theorem. For details, see here.
Solution
I will use a lemma.
Lemma.
If \triangle DEF is the intouch triangle of \triangle ABC, and if I is the incentre of \triangle ABC, and if \triangle A'B'C' is the triangle directly homothetic to ABC such that A'\in IA, B'\in IB, C'\in IC; then A'B'C' is perspective to \triangle DEF.

Now let us come back to our original problem.
Let DEF be, as usual, the intouch triangle of \triangle ABC. Then let \Omega be the incircle of \triangle ABC. Since XD=DI, we see that the polar of X with respect to \Omega is the perpendicular bisector of \overline{DI}. Similarly, the polar of Y w.r.t \Omega is the perpendicular bisector of \overline{IE} and the polar of Z w.r.t \Omega is the perpendicular bisector of \overline{IF}.
Let these perpendicular bisectors intersect each other to form a triangle KLM.
Denote by \omega_1, \omega_2, \omega_3; the circles (D,DI), (E,EI),(F,FI). Then K is the intersection of LK and KM. Since LK is the radical axis of \Omega and \omega_3; and since KM is the radical axis of \Omega and \omega_2, therefore K is the radical centre of \omega_2,\omega_3,\Omega.
Since AE^2-r^2=AF^2-r^2 where r is the inradius of ABC, we note that A has the same power with respect to \omega_2 and \omega_3. So A,K,I all lie on the radical axis of these two circles, and therefore, collinear.
From here it is also obvious that AK=KI,\ BL=LI,\ CM=MI and so \triangle ABC and \triangle KLM are directly similar and perspective around I. So they are homothetic.
Applying the aforementioned lemma, we note that the intersections U=LM\cap EF,\ V=MK\cap FD, \ W=KL\cap DE are collinear.
Since AE and AF are tangents to \Omega, we again note that A is the pole of EF w.r.t \Omega. Also since the polar of X w.r.t \Omega is LM, it follows from the La Hire’s theorem that the pole of AX w.r.t \Omega is U.
Due to symmetry we note that V,\ W are respectively the poles of BY, \ CZ. We already have seen that U,V,W are collinear. Therefore it is obvious that AX,BY,CZ meet at the pole of \overline{VUW} w.r.t \Omega. We are done. \Box
Image

\boxed{E7}.(Polish MO Second Round 2012) Let ABC be a triangle with \angle A=60^{\circ} and AB\neq AC, I-incentre, O-circumcentre. Prove that perpendicular bisector of AI, line OI and line BC have a common point.
Solution
Assume that the incircle touches BC, CA, AB at D,E,F and that X=AI\cap \odot(DEF). Since \angle FIA=60^{\circ},\angle EIA=60^{\circ}, therefore XEI and XFI are equilateral. Also \angle XFA=30^{\circ}=\angle FAX, therefore AX=XE=IX, and the incircle of ABC bisects AI.
Now, we claim that OI is the perpendicular bisector of DX.
Proof of claim.
Let I' be the reflection of I in BC. Then note that we have \angle BI'C=\angle BIC=120^{\circ}, therefore I' lies on \odot(ABC). Now we already have AI=II'=2r, \ I'O= AO, which give OIA\cong OIA', leading to the fact that OI bisects AI'. This readily implies that OI bisects DX.
import graph; size(15cm); real labelscalefactor = 0.5; /* changes label-to-point distance */pen dps = linewidth(0.7) + fontsi...
Coming back to our main proof, note that we have IO as the perpendicular bisector of DX, so OI passes through the pole of DX wrt \odot(DEF). Since pole of DX is XX\cap BC, and because XX is the perpendicular bisector of AI, we are done. \Box

\boxed{E8}.(2012 Indonesia Round 2 TST 4: P2) Let \omega be a circle with center O, and let l be a line not intersecting \omega. E is a point on l such that OE is perpendicular with l. Let M be an arbitrary point on M different from E. Let A and B be distinct points on the circle \omega such that MA and MB are tangents to \omega. Let C and D be the foot of perpendiculars from E to MA and MB respectively. Let F be the intersection of CD and OE. As M moves, determine the locus of F.
Solution(hatchguy)
Let E' be the inverse of E wrt \omega. I claim F is the midpoint of EE'. Clearly, since M is in the polar of E' then E' is in the polar of M, and therefore A,E',B are collinear. It is easy to see that M,E,B,O,A lie on a circle with diameter MO. From this, it’s very natural to think of droping a perpendicular from E to AB. Let G be the foot of this perpendicular. By Simson’s line theorem, we have C,D,F,G are collinear. Also, using that DEGB is cyclic we easily get \angle FGE = \angle DBE = \angle MOE = \angle FEG;The last because MO \parallel EG. Hence FG= FE and we get that F is midpoint of EE' since EGE' is a right triangle. We are done.
import graph; size(15cm); real labelscalefactor = 0.5; /* changes label-to-point distance */pen dps = linewidth(0.7) + fontsi...
\Box

\boxed{E9}. Let incircle \omega of \triangle ABC be tangent to BC , CA , AB in D, E , F. A line from A that is parallel to DE meets DF in K and a line from A that is parallel to DF meets DE in L. If M,N be midpoint of AB , AC , show that KL is on MN.
Solution
From simple angle chasing, we see that \angle DEN=\angle KAE=\angle KFE. This leads to the fact that A,F,K,I,L,E lie on a circle with AI as diameter. Thus, IK\perp KA; leading to C,I,K being collinear. Let H be the orthocentre of \triangle BIC. Then note that polar of K passes through the pole of DF wrt \omega, which is B. Also since IK'\perp DE, it follows that K'=BH\cap CI. Similarly, L'=CH\cap BI. Therefore, the pole of KL wrt \omega is H. Thus, HI\perp KL, and HI\perp BC, leading to KL\parallel MN.
Let L_1=AL\cap BC, then note that we have BAL_1 is B- isosceles, and because BI\perp AL_1, therefore L is the midpoint of L_1A. Thence ML\parallel BC. Similarly NK\parallel BC. Using all these, we see that M,N,K,L lie on a line parallel to BC.
import graph; size(15cm); real labelscalefactor = 0.5; /* changes label-to-point distance */pen dps = linewidth(0.7) + fontsi...
Result:
This problem teaches us that the polar of the orthocentre of BIC wrt \omega bisects AB and AC. This is a very useful fact. Don’t ask me why, because even I don’t know where it may come in handy. :) \Box

\boxed{E10}.(Romania MOM 2012)Let ABC be a triangle and let I and O denote its incentre and circumcentre respectively. Let \omega_A be the circle through B and C which is tangent to the incircle of the triangle ABC; the circles \omega_B and \omega_C are defined similarly. The circles \omega_B and \omega_C meet at a point A' distinct from A; the points B' and C' are defined similarly. Prove that the lines AA',BB' and CC' are concurrent at a point on the line IO.

Solution(r1234)
Firstly, we’ll cite a lemma:
Lemma.
Let \ell be a line and \Gamma be circle.Suppose P is the pole of \ell wrt \Gamma.Now let \triangle ABC be inscribed in \Gamma.Let \triangle A'B'C' be the circumcevian triangle of \triangle ABC wrt P.Then \ell is the perspective axis of \triangle ABC and \triangle A'B'C'.
Proof of lemma.
Let A_1=BC\cap B'C', B_1=AC\cap A'C', C_1=AB\cap A'B'.
Now applying Pascal’s theorem on the hexagon B'C'ABCA' we get A_1,C_1, C'A\cap CA' are collinear.
Again \triangle AC'B and \triangle A'CB' are perspective.So C_1, C'A\cap CA', BC'\cap B'C are collinear.Hence we conclude that A_1,C_1, BC'\cap B'C are collinear.But this line is nothing but the polar of P i.e \ell.Hence we conclude that A_1B_1C_1\equiv \ell.\Box

Coming back to the main proof,
Clearly AA' is the radical axis of \omega _b, \omega_c, BB' is the radical axis of \omega_c, \omega_a and CC' is the radical axis of \omega_a, \omega_b.So by radical axis theorem AA', BB', CC' are concurrent.
Let (I) be the incircle of \triangle ABC and \triangle DEF is its intouch triangle.D', E', F' are the touch points of (I) with \omega_a, \omega_b, \omega_c respectively.Now let tangents at D', E', F' meets BC, CA, AB at X,Y,Z respectively.Then its easy to show that XYZ is the radical axis of (I) and \odot ABC.So X is the pole of DD' wrt (I) and similar for others.So DD', EE', FF' concur at the pole of XYZ wrt (I).Now consider the circles (I), \omega_b, \omega_c.Then by radical axis theorem the lines AA', E'E', F'F' are concurrent ,say at X_1.Then X_1 is the pole of E'F' wrt (I).Since A, X_1, A' are collinear, their polars i.e EF,E'F',\text{Polar of A'} are concurrent.So \triangle DEF and the triangle formed by the polars of A', B', C' are perspective wrt the perspective axis of \triangle DEF, \triangle D'E'F'.But according to our lemma this perspective axis is the polar of the perspective point of \triangle DEF, \triangle D'E'F', i.e the radical axis of (I), \odot ABC.So we conclude that AA', BB', CC' concur at the pole of the radical axis of (I), \odot ABC wrt (I).Since IO\perp \text{Radical axis of (I),circumcircle of ABC} we conclude that the pole of the radical axis of (I), \odot ABC lies on IO.Hence AA', BB', CC' concur on IO.
import graph; size(15cm); real labelscalefactor = 0.5; /* changes label-to-point distance */pen dps = linewidth(0.7) + fontsi...
\Box

As for exercises, I won’t give any for this post. I will just write down some references from where you can get plenty of problems. ^_^

References.
1. Cyclic Quadrilaterals – The Big Picture by Yufei Zhao.
2. Power of a Point by Yufei Zhao.
3. Circles by Yufei Zhao.
4. Poles and Polars by Kin Y. Li.
5. Mathscope Topic on Poles and Polars by Hoàng Quốc Khánh.
6. Introduction to the geometry of triangle by Paul Yiu.

A digression on Calculus… From introductory to advanced?

Before anything, wish everyone a happy and prosperous new year full of joy and success! :) (This was posted on Dec 31)

Surprised, aren’t you? Well, you may not be expecting this, but I came across a bunch of beautiful problems. So I want to introduce a little bit of graphs (is that really necessary?).
Without wasting any more time, let’s continue with this hotch-potch discussion on Calculus. Yeah, I know that I am not organised, sorry about that.
I don’t know from where to start, so anyway, let me start with the fundamentals of calculus problem-solving. The problems that we(of course, ‘we’ means that I am going to follow the Indian curriculum of Calculus) usually solve consist of the following concepts: Graphs and Functions, Limits, continuity and differentiability, function-plotting.
So anyway, let me start with some basic concepts of Polynomials.
Polynomials.
These may be defined as functions (that’s my way of looking at them) which have terms of integral powers and real or complex coefficients. For the sake of calculus of our level though, we only look at polynomials P[\mathbb R](x) which is generally given by
P(x)\equiv a_0x^n+a_1x^{n-1}+\cdots+a_n; given a_i\in\mathbb R\forall i=1,2,\cdots n.
The degree of a polynomial \deg{P(x)} is the highest power of x that is contained in P. If a_0\neq 0, then the degree of P is n.
Polynomials of even degree
If n is even, then we can only have two types of graphs possible.
P(x)=a_0x^n+a_1x^{n-1}+\cdots+a_n=x^n\left(a_0+\frac{a_1}{x}+\cdots+\frac{a_n}{x^n}\right).
Assume that x\to\infty, so that \lim_{x\to\infty}P(x)=\lim_{x\to\infty}a_0x^n=\left\{\begin{aligned}&+\infty, a_0>0\\&-\infty, a_0<0.\end{align...
And, \lim_{x\to-\infty}P(x)=\lim_{x\to-\infty}a_0x^n=\left\{\begin{aligned}&+\infty, a_0>0\\&-\infty, a_0<0.\end{ali...
So the graph can be of two different looks:
import graph; unitsize(1cm);size(8cm);real f(real x) {return (36-27x/100-13x^2/10000+3x^3/1000000+x^4/100000000);}pair F(real...
import graph; unitsize(1cm);size(8cm);real f(real x) {return (-x^4/100000000+200 x^3/100000000+370000 x^2/100000000-86000000 ...
So, a polynomial with even degree will either rise to the heaven, or fall down to hell on both sides – x\to\infty and x\to-\infty. It’s also obvious that it will cut the x axis even number of times.
Polynomials of odd degree
It’s easy to deduce that the graphs of polynomials with odd degree will fall to hell at one side, and rise up to heaven on the other. A nice way of recalling is that, if a_0>0, then the graph will rise up towards heaven at x\to\infty and vice versa. Like,
import graph; unitsize(1cm);size(8cm);real f(real x) {return (x^3/1000000+x^2/5000-(29 x)/100-30);}pair F(real x) {return (x,...
import graph; unitsize(1cm);size(8cm);real f(real x) {return (-x^3/1000000-x^2/5000+(29 x)/100-30);}pair F(real x) {return (x...
Roots
The roots of a polynomial are the points where the curve cuts the x-axis. We can use Descartes sign rule to determine the number of positive or negative roots. Let P(x) be a certain polynomial, which has 'n' number of sign changes as while proceeding from the lowest to the highest power (ignoring zero coefficients), has a maximum of n positive roots. It may have n-2, n-4 etc positive roots too. When this rule is applied to P(-x), we can guess the maximum allowed negative roots.

The derivative
Let f(x) be a function. Then f'(x_0) is the derivative at a point x_0, which is defined as f'(x_+)=\lim_{h\to 0}\frac{f(x+h)-f(x)}{h}, f'(x_-)=\lim_{h\to0}\frac{f(x)-f(x-h)}{h}. If f'(x_+)=f'(x_-)=f'(x) in an interval (a,b), then the function is said to be differentiable over all points in (a,b). Then f'(x_0) is the slope of the tangent to the curve f(x) at x=x_0. A function is differentiable only if it’s continuous, but the reverse is not true.
Again, I do not intend on lecturing on removable and blah blah continuity, so I am assuming prior knowledge without loss of generality. After all, it’s tiring to mention everything, lol. So let’s get into some serious stuff.

Convexity, concavity, monotonicity.
A function is said to be monotonically, or strictly increasing, or decreasing if and only if f(a)\geq f(b), f(a)>f(b) and its reverses respectively hold for any a,b in a certain interval.
A function f(x) defined on an interval is called convex (or convex downward/concave upward) if the graph of the function lies below the line segment joining any two points of the graph. The formal definition is, a function f:\mathbb I\to\mathbb R is convex if and only if for any two points x_1,x_2\in\mathbb I and \lambda\in(0,1) we have f(\lambda x_1+(1-\lambda)x_2)\leq \lambda f(x_1)+(1-\lambda)f(x_2). In case the equality doesn’t occur, the function is called to be strictly convex.
Concavity is just the opposite.
The necessary and sufficient condition for a certain function f(x) to be convex in an interval (a,b) is that the function must lie above all of its tangents, ie f(x)\geq f(y)+(x-y)f'(y). Concavity: reverse the sign. :D
This may also be rephrased as f''(x)\geq 0 for all x in \mathbb I. We may show that this is another necessary and sufficient condition for convexity.
Jensen’s inequality.
Let f be a convex function of one real variable. Let x_1,\cdots,x_n\in\mathbb R and let a_1,\cdots, a_n\ge 0 satisfy a_1+\dots+a_n=1. Then
f(a_1x_1+\cdots+a_n x_n)\le a_1f(x_1)+\cdots+a_n f(x_n).
For a proof, look here.

Without any more of introduction, let me move on to one of the most important theorems in Calculus, ie Rolle’s theorem.
Rolle’s Theorem
If f(x) is continuous and differentiable at every point in an interval [a,b], and if f(a)=f(b), then there exists a certain x_0\in[a,b] such that f'(x_0)=0.
From graphical considerations, it is obvious that the function must have changed its slope from either positive to negative, or negative to positive at some point x_0 in the given interval. It can contradict this at the cost of its differentiability everywhere.

Mean Value Theorem (MVT for short)
If f(x) and g(x), and f'(x), g'(x) are continuous throughout an interval [a,b], and if f'(x)\neq 0 everywhere in the given interval, then there will always exist a point x_0\in[a,b] such that
\frac{f(b)-f(a)}{g(b)-g(a)}=\frac{f'(x_0)}{g'(x_0)}.
Proof.
Consider the function \phi(x)=\frac{f(b)-f(a)}{g(b)-g(a)}[g(x)-g(a)]-[f(x)-f(a)].
Since \phi(a)=\phi(b)=0, so applying Rolle’s theorem we see that
\phi'(x)=\frac{f(b)-f(a)}{g(b)-g(a)}g'(x)-f'(x)=0 for some point x=x_0. Hence done.
Geometric interpretation
For the non-general version of Rolle’s theorem for g(x)=x, we can see that the slope of the line joining (a,f(a)) and (b,f(b)) is \frac{f(b)-f(a)}{b-a}, and f'(x) is the slope at x=x. So, obviously there exists a certain x_0 in the interval such that f'(x_0) equals the slope of the chord joining the terminal points.
import graph; unitsize(1cm);size(8cm);real f(real x) {return (x^3/1000-2x^2/100+2x/10+1);}pair F(real x) {return (x,f(x));}xa...
Extended MVT
Define a constant R such that the equation
f(b)-f(a)-(b-a)f'(a)-\frac 12(b-a)^2R=0
Is satisfied. Define a function F(x) as
F(x)=f(x)-f(a)-(x-a)f;(a)-\frac 12 (x-a)^2R.
Since F(a)=F(b)=0, therefore F'(x)=f'(x)-f'(a)-(x-a)R, and
F'(x_1)=f'(x_1)-f'(a)-(x_1-a)R=0 for some x_1\in[a,b].
Since F'(a)=F'(b)=0, therefore \exists x_2\in[a,b]; \ni F''(x_2)=0, and R=f''(x_2). Substituting R, we get
f(b)=f(a)+(b-a)f'(a)+\frac{(b-a)^2}{2!}f''(x_2) \ \ (x_2\in (a,b)).
Continuing, we obtain,
\begin{aligned}f(b)=f(a)+\frac{(b-a)}{1!}f'(a)+\frac{(b-a)^2}{2!}f''(a)+\cdots+\frac{(b-a)^n}{n!}f^{(n)}(a)\\ +\frac{(b-a)^{n...
Where x_1\in(a,b). This expression is known as the extended MVT.

With this, I am wrapping up all my discussions on the theorems that we study in high school (except for the extended MVT, everything is quite fundamental).
And, it also means that I will now move onto some nice problems. Most of the problems that I will discuss in this post will be involving construction of functions and applications of Rolle’s theorem, etc. So, let’s change the mood from theoretic to a little of applications.
Problems.
1. Given two functions f and g, continuous on [a, b], differentiable on (a, b), and f(a)=f(b) = 0. Prove that there exists a point c\in(a, b) such that g'(c)f(c) + f'(c) = 0.
Solution
Define h(x)=f(x)e^{g(x)}, then h(a)=h(b)=0 and so, there exists a c in (a,b) such that h'(c)=0. But, note that h'(c)=e^{g(x)}\left(f'(x)+f(x)g'(x)\right)=0, and this leads us to our desired result.\Box

2. Let f be a function from reals to reals with at least two roots a<b . Prove that for any real number k there is c\in (a,b) such that f(c)+kf'(c)=0 .

Solution(Virgil Nicua)
If k=0 , then can choose c\in\{a,b\} . Suppose k\ne 0 . In this case consider g:[a,b]\rightarrow \mathbb R, Where g(x)=e^{\frac xk}\cdot f(x) . Observe that g satisfies g(a)=g(b)=0 and g'(x)=e^{\frac xk}\cdot\left[f'(x)+\frac 1k\cdot f(x)\right], so by Rolle’s theorem we get our desired result. \Box

3. Show that for two functions f,g such that g(x)\neq 0\forall x\in\mathbb R, and given s,r\in\mathbb R, we have a certain c between any two roots a,b of f such that
\frac{f'(x)}{g'(x)}=\frac{s}{r}\cdot\frac{f(x)}{g(x)}.
Solution
Consider h(x)=\frac{f(x)^r}{g(x)^s}. Indeed, h(a)=h(b)=0, so there exists at least one c between a,b such that
\frac{\text{d}}{\text{d} \ x}\left(\frac{f(x)^r}{g(x)^s}\right) = \frac{f(x)^{r-1}}{g(x)^{s+1}}(r\cdot g(x)f'(x)-s\cdot f(x)g...
Which leads to our desired result.\Box

4. Solve in \mathbb{R}, the following equation:
2^x-x=1.
Solution
Though it’s obvious from graphs that x=0,1 are the only solutions, why not apply Rolle’s? Suppose f(x)=2^x-x-1 has at least three distinct roots. By Rolle’s theorem, f'(x)= 2^x \log 2 - 1 should have at least two distinct roots; however f' is strictly increasing and so this cannot happen. So, f has at most two distinct roots, namely 0 and 1.

5. (Amparvardi) Let f: [a,b] \to \mathbb R, 0<a<b be a function which is continuous and differentiable in (a,b). Prove that there exists a real number c \in (a,b) for which
f'(c)=\frac1{a-c}+\frac1{b-c}+\frac1{a+b}.
Solution
Let us assume \displaystyle g(x)=(x-a)(x-b)\cdot e^{f(x)-\frac x{a+b}}. Note that g(a)=g(b)=0, and also
\begin{aligned}g'(x)&={e^{f(x)-\frac x{a+b}}}\cdot\left(2x-a-b-\frac{(x-a)(x-b)}{a+b}+(x-a)(x-b)f'(x)\right)\\&=(x-a)...
So from Rolle’s theorem, there exists at least one c\in(a,b) such that g'(c)=0, and we are done.\Box

Let’s see some LMVT problems now.
6. Show that

\frac{x-1}{x} \leq \ln x \leq x-1   for    x\in(0,\infty).
Solution
Consider the function f(x)=\ln x , it is continuous in [1,x] and also differentiable in (1,x). Applying LMVT, there must be one c such that f'(c) =\frac{\ln x}{x-1}
But 1 \leq c \leq x , therefore \frac{1}{x} \leq \frac{1}{c } \leq 1 \implies \frac{1}{x} \leq \frac{\ln x}{x-1} \leq 1, and hence we get our desired inequality.

7. Prove that for any x\in\left(0,\frac{\pi}{2}\right), we have
\left(\frac x{\sin x}\right)^{\tan x-x}>\left(\frac {\tan x}{x}\right)^{x-\sin x}.
Solution(Virgil Nicua)
Since 0<\sin x<x<\tan x, we can rewrite the given problem into
\frac{\ln x-\ln \sin x}{x-\sin x}>\frac{\ln\tan x-\ln x}{\tan x-x}.
Apply LMVT to the function \ln:(0,\infty)\to\mathbb R on the segments [\sin x, x] and [x,\tan x]. So there exist 0<\sin x<c<x<d<\tan x satisfying \frac{\ln x-\ln \sin x}{x-\sin x}=\frac 1c and \frac{\ln\tan x-\ln x}{\tan x-x}=\frac 1d. So, 0<c<d\implies
\frac{\ln x-\ln \sin x}{x-\sin x}>\frac{\ln\tan x-\ln x}{\tan x-x}\ \ \ \forall x\in \left(0,\frac {\pi}{2}\right).\Box

8. Let p(x)=ax^2+bx+c be a polynomial with real roots. Given that a and 24a+7b+2c have the same sign, prove that it’s impossible for both roots of p(x) to be in the interval (3,4).
Solution(marvopnema)
Assume the two roots of p(x) are both real, and situated in (3,4), hence of the form 3+u, 3+v, with 0<u,v<1. From Vieta’s relations we have -\dfrac {b} {a} = (3+u) + (3+v) = 6 + (u+v) and \dfrac {c} {a} = (3+u)(3+v) = 9 + 3(u+v) + uv.
Then \dfrac {1} {a} (24a + 7b + 2c) = 24 - 42 - 7(u+v) + 18 + 6(u+v) + 2uv = 2uv - (u+v) \leq 2uv - 2\sqrt{uv} = 2\sqrt{uv}(\sqrt{uv} - 1) < 0, hence a and 24a+7b+2c are of different signs.\Box

9. While a>0 and n\in\mathbb N, find the limit
\lim_{n\to\infty}n^2\left(\sqrt[n]{a}-\sqrt[n+1]{a}\right).
Solution(hsbatt)
By LMVT,
\begin{aligned}n^2 \left(a^{\frac{1}{n}} - a^{\frac{1}{n+1}} \right) &= n^2 a^{\frac{1}{t}} \ln a \left( \frac{1}{n}-\fra...
For some t \in (n,n+1).
Now, let’s sandwich the expression between \frac{n^2 a^{\frac{1}{n}} \ln a }{n(n+1)} and \frac{n^2 a^{\frac{1}{n+1}} \ln a }{n(n+1)}
Because both the limits of the left and the right bound are \ln a, therefore the function bounded in-between must also have a limit \ln a and n\to \infty.\Box

10. Without integrating, show that the sum

;S_n=\sum_{k=1}^n\frac 1{k\sqrt k}

converges.
Solution
Consider f:[k,k+1]\to\mathbb R, where f(x)=\frac 1{\sqrt x} and k=1,2,\cdots n-1. Observe that f'(x)=-\frac 1{2x\sqrt x}. By LMVT, there must exist a certain c\in[k,k+1] such that f(k+1)-f(k)=f'(c). Since f' is increasing, we get
f'(k)<f(k+1)-f(k)<f'(k+1);
Or,
\frac 1{2k\sqrt k}>\frac 1{\sqrt k}-\frac 1{\sqrt{k+1}}>\frac 1{2(k+1)\sqrt{k+1}}.
Summing up from 1 to n, we get
S_n<2\left(1-\frac 1{\sqrt{n+1}}\right)>S_n-1+\frac1{\sqrt{n+1}};
Or,
2\left(1-\frac 1{\sqrt{n+1}}\right)<S_n<3\left(1-\frac 1{\sqrt{n+1}}\right).
So, the sequence is increasing and bounded above and below. So we see that 2-\sqrt 2\leq S_n\leq 3, and S_n converges.\Box

11. Let p(x)=x^n+a_{n-1}x^{n-1}+\cdots+a_1x+a_0 be a monic polynomial of degree n>2, with real coefficients and all its roots real and different from zero. Prove that for all k=0,1,2,\cdots,n-2, at least one of the coefficients a_k,a_{k+1} is different from zero.
Solution
Lemma 1
If all the roots of p are real, then all the roots of p' are real.
* The proof of this lemma is clear, because between two consecutive distinct roots of p there is a root of p' (using Rolle’s theorem), and we have that if r is a root of p with multiplicity m\geq 2, then it’s a root of p' with multiplicity m-1.
Lemma 2
At least one of a_1,a_2 is different from 0
* Suppose that the (possibly repeated) roots of p are r_1,r_2,\ldots,r_n, and that a_1=a_2=0.
Then from Viete’s formulas we get that:
\begin{cases}a_1=r_1r_2\ldots r_n\left(\frac{1}{r_1}+\ldots\frac{1}{r_n}\right)=0\\a_2=r_1r_2\ldots r_n\left(\frac{1}{r_1r_2}...
As the r_i are nonzero, this immediately implies that \frac{1}{r_1^2}+\ldots+\frac{1}{r_n^2}=\left(\frac{1}{r_1}+\ldots\frac{1}{r_n}\right)^2-2\left(\frac{1}{r_1r_2}+\frac{1}{r_1r..., a clear contradiction. This concludes the proof of lemma 2. \Box
We now prove that the statement is true by strong induction on n\geq 1:
Base cases:n=1,2
If n=1 then there is nothing to prove, and if n=2 then the result is true because a_0\neq 0 (because 0 is not a root).
Induction step: Assume that n\geq 3.
Case 1: a_1\neq 0
In this case we have that p' is a polynomial with all roots real and nonzero (using lemma 1), so we may use the induction hypothesis on \frac{p'(x)}{n} (we divide by n just to make the polynomial monic, a minor technicality to allow us to use the hypothesis). As the coefficients of this polynomial are nonzero multiples of the coefficients of p (except a_0), we conclude that there are no consecutive zero coefficients in p.
Case 2: a_2\neq 0
In this case we have that p'' is a polynomial with all roots real and nonzero (using lemma 1 twice), so we may use the induction hypotheses on \frac{p''(x)}{n(n-1)} (again, dividing by n(n-1) is just a technicality), because n-2\geq 1. As the coefficients of this polynomial are nonzero multiples of the coefficients of p (except a_0,a_1), again we conclude that there are no consecutive zero coefficients (using lemma 2 for the coefficients a_1,a_2).
This concludes the proof of the desired statement. \Box

12. (a) Prove that

\sum_{k=0}^{2n-1}{\frac{x^{k}}{k!}

has only one real root x_{2n-1}.
(b) Prove that x_{2n-1} is decreasing, and \lim_{x\rightarrow\infty}{x_{2n-1}}=-\infty.
Solution to Part (a)
Let

P_{m}(x)=\sum_{k=0}^{m}\frac{x^{k}}{k!}
The key to proving that P_{2n-1} has exactly one real root is to show that P_{2n} has no real roots.
Note that for x\ge0,\ P_{m}(x)\ge 1. Hence all zeros must occur for negative x
By Taylor’s Theorem (with Lagrange mean-value form of the remainder),
e^{x}=P_{2n}(x)+\frac{e^{\xi}x^{2n+1}}{(2n+1)!}
But for x<0,\ \frac{e^{\xi}x^{2n+1}}{(2n+1)!}<0 which implies that P_{2n}(x)>e^{x}>0.
Hence, P_{2n}(x) has no zeros. Now consider P_{2n+1}(x). As an odd degree polynomial, it must have at least one zero. However, by Rolle’s Theorem, if it has two or more zeros, then its derivative must have at least one zero. But P_{2n+1}'(x)=P_{2n}(x), which has no zeros. Hence, P_{2n+1}(x) has exactly one zero.
Now, given any B<0, we know that P_{m}(x) tends to e^{x} uniformly on [B,0]. This implies that there exists an N, depending on B, such that for m>N,\ P_{m}(x) has no zeros in [B,0]. This establishes that the sequence of roots, x_{2n-1}, must tend to -\infty as n\to\infty.
Solution to Part (b)
Let Q_{k}(x) = \frac{x^{2k}}{(2k)!}+\frac{x^{2k+1}}{(2k+1)!}= \frac{x^{2k}}{(2k+1)!}\cdot \left( x+2k+1 \right).
We have that P_{2n-1}(-2n-1) = Q_{0}(-2n-1)+Q_{1}(-2n-1)+\ldots+Q_{n-1}(-2n-1).
Note that Q_{k}(-2n-1) < 0, \, \forall k \in \overline{0,n-1}, since (-2n-1)+2k+1=2(k-n) < 0.
Hence, P_{2n-1}(-2n-1) < 0. Since P_{2n-1} is a polynomial of odd degree and has a unique root, we must have that
-2n-1 < x_{2n-1}.
Consequently,
\begin{eqnarray*}P_{2n+1}\left( x_{2n-1}\right) &=& P_{2n-1}\left( x_{2n-1}\right)+Q_{n}\left( x_{2n-1}\right) \\ \ &...
Since P_{2n+1} is a polynomial of odd degree and has a unique root, we must have that x_{2n+1}< x_{2n-1}, i.e. \left\{ x_{2n-1}\right\}_{n \geq 1} is strictly decreasing.\Box

And, I finish the post here only. 12 Problems for this post!
Also, sorry for not being able to normalise. The introduction part is too easy, and the last problem uses Taylor. :lol:

A nice Functional Equation

This post was originally posted by: Rijul Saini

I solved a cute functional equation today, and was also wondering what I was doing being a contributor here, so that resulted in this.
IMO Short List 2002 A01
Find all functions f: \mathbb{R} \rightarrow \mathbb{R} such that

f\left(f(x)+y\right)=2x+f\left(f(y)-x\right)

for all real x,y

Solution

Let P(x,y) denote the assertion f(f(x)+y) = 2x + f(f(y)-x).
Now,
P(0,x) \implies f(x+f(0)) = f(f(x))
P(f(y),y) \implies f(f(f(y)) + y ) = 2f(y)+f(0) \implies f(f(y+f(0))+y) = 2f(y) + f(0)Taking y=-f(0) here, implies f(0) = 2f(-f(0)) +f(0) \implies f(-f(0)) = 0.

Next,
P(-\frac x2, -f(-\frac x2) - f(0)) \implies x = f(f(-f(-\frac x2) - f(0)) + \frac x2)

Therefore, f is onto i.e. covers all values of x.

Now,let f(0) = c, and define a function g : \mathbb{R} \rightarrow \mathbb{R} as g(x) = f(x - c).

Therefore, our original assertion P(x,y) for f changes into the equivalent assertion Q(x,y) for g, as being
g(g(x+c) + y + c) = 2x + g(g(y+c) - x +c)

As f is onto, therefore g is also onto. Also, g(0) = 0. And then finally,

Q(-c,x-c) \implies g(x) = -2c + g(g(x) + 2c) \implies g(g(x) + 2c) = g(x) + 2c

As g is onto, therefore, g(x) covers all values of x, and therefore, g(x) + 2c also covers all values of x,

Thus, g(x) = x for every x \in \mathhbb{R}.

And therefore, f(x) = x+f(0) for every x \in \mathbb{R} and this is indeed a solution.

Harmonic Divisions – A Powerful & Rarely Used Tool!

Long since I have ever posted anything on my blog, and more importantly, I have not posted anything on geometry before (except for a similarity trivia), and therefore this is my first extensive attempt on geometry.
As the title explains clearly, I want to post some useful results and solve some problems related to Harmonic Divisions, because I have not found a very good book or ebook explaining the concepts clearly. :)
HARMONIC DIVISIONS
Though all of us know what a harmonic division is, if four points A,B,C,D are collinear and if they satisfy
\frac{AC}{CB}=\frac {AD}{DB}, then the four points ACBD are said to form a harmonic range.
Now let us move onto some theorems.
Theorem 1.
If (ACBD) is a harmonic range and if O is the midpoint of AB then OB^2=OC\cdot OD.
Proof.
After componendo and dividendo on \frac{AC}{CB}=\frac{AD}{DB}; we get \frac{AC+CB}{AC-CB}=\frac{AD+DB}{AD-DB}; leading to
\frac{2OB}{2OC}=\frac{2OD}{2OB}\iff OB^2=OC\cdot OD.
The converse of this theorem is also obviously true.
dot((0,0));dot((-5,0));dot((+5,0));dot((3,0));dot((8,0));draw((-5,0)--(8,0));label("$A$", (-5,0), S);label("$O...
Theorem 2.
If AX, BY, CZ are three concurrent cevians of a triangle \triangle ABC with X, Y, Z lying on BC, CA, AB respectively, then ZY\cap BC=T\implies (BXCT) is a harmonic division.
Proof
Using Menelaus we have,
\frac{|AZ|}{|BZ|}\cdot\frac{|BT|}{|CT|}\cdot \frac{|CY|}{|AY|}=1;
And using Ceva we obtain
\frac{|AZ|}{|BZ|}\cdot\frac{|BX|}{|CX|}\cdot\frac{|CY|}{|AY|}=1;
Which jointly lead to \frac{|BX|}{|CX|}=\frac{|BT|}{|CT|}\implies (BXCT)=-1; ie the division is harmonic.
size(10cm, 8cm);label("$A$", (0,5), N);label("$B$", (-1,0), S);label("$C$", (5,0), S);label(&qu...
Theorem 3.
If four concurrent lines are such that one transversal cuts them harmonically, then every transversal intersects these four lines to form a harmonic division.
Proof
Assume the lines OA, OB, OC, OD pass through O and cut by a line d in A,B,C,D respectively. Assume that (ACBD) is harmonic, and therefore let another transversal cut these at A',B',C',D'; it is sufficient to check that the four points A', B', C', D' form a harmonic division.
Note that using the sine rule we get
\frac{AC}{CB}=\frac{OA}{OB}\cdot\frac{\sin\angle AOC}{\sin\angle COB}; and \frac{AD}{BD}=\frac{OA}{OB}\cdot\frac{\sin\angle AOD}{\sin\angle BOD}.
Comparing these two and using the fact that (ACBD) is harmonic, we get \frac{\sin\angle AOC}{\sin\angle COB}=\frac{\sin\angle AOD}{\sin\angle BOD}.
Now again we can use similar arguments to arrive at \frac{A'C'}{C'B'}=\frac{A'D'}{B'D'}; ie (A'C'B'D') is harmonic.
import graph; size(300); real lsf = 0.5; pen dp = linewidth(0.5) + fontsize(10); defaultpen(dp); pen ds = black; pen xdxdff =...
Note: Such four concurrent lines are called to form a harmonic range or a harmonic pencil, denoted as O(ACBD) or sometimes as O(AB; CD).
Theorem 4.
Any two of the following statements implore the third statement to be true, too.
Statement 1: The pencil O(AB; CD) is harmonic.
Statement 2: OB bisects \angle AOC internally.
Statement 3: OB and OD are perpendicular to each other.
Proof.
I will prove that (1), (3)\implies (2), and the rest is left for the reader to prove.
Refer to the diagram. Draw EF\parallel AO such that E\in OC, F\in OD. Now,
AO\parallel EF\implies \frac{AC}{CB}=\frac{AO}{EB}; and AO\parallel BF\implies \frac{AD}{BD}=\frac{AO}{BF}.
Using \frac{AC}{CB}=\frac{AD}{BD}, we get EB=BF, ie B is the midpoint of EF.
Now, we already have \angle OBF=\angle BOA=\frac{\pi}{2}\land EB=BF\implies \triangle OBE\cong\triangle OBF. Therefore OB bisects \angle EOF, but since this is perpendicular to OA, therefore OA is the external bisector of \angle EOF.
import graph; size(300); real lsf = 0.5; pen dp = linewidth(0.5) + fontsize(10); defaultpen(dp); pen ds = black; pen xdxdff =...
Theorem 5.
If (AB; CD) and (A'B'; C'D') are harmonic, and if AA', CC', BB' concur, then the line DD' is also concurrent with these lines.
Proof.
Let OD' meet BC at some point D'. We obtain that O(A'B'C'D') is harmonic, implying (AB, CD') is also harmonic. So we get
\frac{AC}{CB}=\frac{AD'}{BD'}. But also \frac{AC}{CB}=\frac{AD}{BD}; so that
\frac{AD'}{BD'}=\frac{AD}{BD}\implies \frac{AD'}{AD'-BD'}=\frac{AD}{AD-BD}\implies AD=AD', so that D' and D are coincident.
Image
Theorem 6. (NEW)
Let ABCD be a convex quadrilateral. If K=AD\cap BC, M=AC\cap BD, P=AB\cap KM, Q=DC\cap KM, then we must have K,M,P,Q to form a harmonic division.
Proof.
Let R=BQ\cap AK. Then from Theorem 2 in \triangle KDC with cevians KQ,CA,BD and transversal RQB, we observe that the division K,R,D,A is harmonic.
So the range B(K,R,D,A) is harmonic and using KP as a transversal, we see that (KMPQ)=-1. We are done.
import graph; size(14cm); real lsf = 0.5; pen dp = linewidth(0.7) + fontsize(10); defaultpen(dp); pen ds = black; pen wwwwff ...
\Box

Theorem 7. (Newer) (BMO 2013)

The point P lies inside triangle ABC so that \angle ABP = \angle P CA. The point Q is such that P BQC is a parallelogram. Prove that \angle QAB =\angle CAP.
Proof.
Assume that P is to the left of Q. Now, complete the parallelogram BASP. Then note that BPQC is also a parallelogram, leading us to the fact that ASCQ is also a parallelogram. Therefore, \angle ABP=\angle ASP=\angle ACP, so that ASCP is a cyclic quadrilateral. Now, \angle QAC=\angle ACS=\angle APS, where the first equality comes from AQ\parallel SC and the second one from the cyclic quadrilateral. So, we are done.

DiagBMO.jpg

\Box
Now we have already done some easy theorems which might come in handy here and there, so we need to keep a tab on them. Let’s move onto some simple applications.
Note: From now on I might denote (ACBD) as the cross-ratio and (ACBD)=-1 obviously denotes the case when (AB; CD) is harmonic.
Note 2: I assume the reader to have some knowledge on Poles and Polars, Inversion, Homothety and Cross ratio. So hope I will not be too ambiguous while using these notations and notions. :lol:

\boxed{E1}. If AD,BE,CF are the altitudes of a triangle \triangle ABC, and if DE, EF meet AB, BC at F', D' respectively, then show that FD and F'D' intersect on a point lying on AC.
Solution
Let FD meet AC in E', then we have to show that D',E',F' are collinear.
Using Theorem 2, we have that (BDCD') is harmonic, ie \frac{BD}{DC}=\frac{BD'}{CD'}.
Similarly we obtain three other relations, and multiplying them out we get
\frac{BD'}{D'C}\cdot\frac{CE'}{E'A}\cdot\frac{AF'}{F'C}=\frac{BD}{DC}\cdot\frac{CE}{EA}\cdot\frac{AF}{FB}\stackrel{\text{Ceva...
So that using the converse of Menelaus theorem, D'E'F' is a transversal to the triangle \triangle ABC, and therefore a straight line.
Note: Even if I used directed segments in this proof, I think it is best to avoid using them.
import graph; size(300); real lsf = 0.5; pen dp = linewidth(0.5) + fontsize(10); defaultpen(dp); pen ds = black; pen wwqqff =...

\boxed{E2}. The tangent at a point P of a circle cuts a diametre AB at T; and PN is the perpendicular to AB, the line joining B to the midpoint of TP cuts PN at Q. Prove that AQ\parallel TP.
Solution
Let us assume QA\cap TP=X. Note that we have, from the similarity of \triangle PBN and \triangle PBA; that
\angle BPT=\angle BAP=\angle NPB. So using Theorem 4, we obtain that (ABTN) is harmonic. So the pencil Q(ABTN) is also harmonic, leading to (suing TX as a transversal) (XMTP) is harmonic.
But, this will be harmonic if and only if \frac{XT}{MT}=\frac{XP}{PM}=\frac{XD}{TM}. But using PM=TM, we get XT=XP, but T\neq P. Therefore X=\infty, and AQ\parallel TP.
import graph; size(300); real lsf = 0.5; pen dp = linewidth(0.7) + fontsize(10); defaultpen(dp); pen ds = black; pen wwqqff =...

\boxed{E4}Let ABC be a right angled triangle at A. D is a point on CB. Let M be the midpoint of AD. CM intersects the perpendicular bisector of AB at E. Prove that BE\parallel DA.
Solution
Let K=AB\cap CE. We will show that if E is a point on CM extended such that BE\parallel DA, Then we must have L is the perpendicular from E onto AB\implies AE=EB.
Now let DA and BE meet at \infty, then; since M is the midpoint of AD it is forced that (DMA\infty) is harmonic.
Now this also implies that B(DMA\infty) is a harmonic range, so that using CE as a transversal to this range it is implored that C,M,K,E are harmonic.
Now since \angle CAK = 90^{\circ} we must have \angle MAK=\angle KAE. Therefore using DA\parallel BE we have \angle DAB=\angle LBE, and therefore \triangle LAE\cong\triangle BEL.
Hence L is the midpoint of AB.\Box
import graph; size(10cm);draw((0,0)--(4,0));draw((4,0)--(0,5));draw((0,0)--(0,5));draw((0,5)--(2,-2.51));draw((0,0)--(2,2.5))...
\boxed{E5}In acute triangle ABC, we have points D and E on sides AC, AB respectively satisfying \angle ADE=\angle ABC. Let the angle bisector of \angle A meet BC at K.\ P and L are projections of K and A to DE, respectively, and Q is the midpoint of AL. If the incenter of \triangle ABC lies on the circumcircle of \triangle ADE, prove that P,\ Q, and the incenter of \triangle ADE are collinear.
Solution(motal)
Let J\equiv AK\cap BD, R the incenter of \triangle ABC and I the incenter of \triangle ADE. (AQL\infty)=-1 so, considering P(AQL\infty) with transversal AK, the problem is equivalent to proving that (AIJK)=-1. Easy angle chasing leads to \triangle ERD isosceles. Furthermore, \angle{ARD}=\angle{AED}=\gamma\Longrightarrow RKCD concyclic. With similar argument we obtain BERK concyclic. \angle{EID}+\angle{EKD}=180^{\circ}-\frac{\beta}{2}-\frac{\gamma}{2}+(\frac{\beta}{2}+\frac{\gamma}{2})=180^{\circ}. Therefore we have that IEKD is concyclic with diameter KI. In particular we have that both of these facts hold: \angle{IDK}=90^{\circ} and DI is bisector of \angle{JDA}. Hence considering pencil D(AIJK) we can conclude that (AIJK)=-1.
We are done. \Box
Sorry I could not attach a figure, I will attach one as soon as possible.

\boxed{E6}.(Desargues) Let \triangle ABC and \triangle A'B'C' be perspective triangles perspective about a point O. If BC\cap B'C'=X, CA\cap C'A'=Y and AB\cap A'B'=Z, then X,Y,Z are collinear.
Solution
Let us denote OA\cap BC=E, OA\cap B'C'=E'. Now we will use a little of cross ratio or projective geometry to prove this fact.
Firstly consider the concurrent points B,E,C,Y and B',E',C',Y. The corresponding lines BB', EE', CC', YY are concurrent at O. Therefore using a simple idea of cross-ratio we obtain that
O(BY, EC)=O(B'Y, E'C). It is also obvious that O(BY,EC)=A(BY, EC)=A(X,Y,Z,O) and O(B'Y,E'C')=A'(X,Y,Z,O). So the points X,Y,Z are collinear since O lies on the line AA'. (Why?)
We are done. :)
Image

\boxed{E7}.Given that AB is a diameter of a circle and the lines CD,CB are tangents to the circle, prove that DE = EF, where F is the foot of perpendicular from D onto AB and E=DF\cap CA.
Solution
Let U=CD\cap BA and S=BD\cap CA.
Since DF\perp AB and BD\perp DA, we see that \angle ADU=\angle ABD=\angle FDA, therefore DA bisects \angle FDU and also AD\perp BD. So the range D(B,F,A,U) is harmonic. Using CA as a transversal this leads to the fact that S,E,A,C are harmonic. So B(S,E,A,C) is harmonic, and again, using DF as a transversal to this range we have that (FED\infty)=-1, ie E is the midpoint of DF.
These types of midpoint considerations can prove a lot of geometric problems easily, and of course this one is obvious from considering symmedians. Apart from that, harmonic divisions stands out to be a hugely useful and rarely used (in my country, at least) method.

\boxed{E8}.(Proposed in Olympiad Marathon by Fersolve)
Let O be the intersection of the diagonals of convex quadrilateral ABCD. The circumcircles
of \triangle OAD and \triangle OBC meet at O and M. Line OM meets the circumcircles of \triangle OAB and \triangle OCD at S and T respectively. Prove that M is the midpoint of ST.(NEW)
import graph; size(14cm); real lsf = 0.5; pen dp = linewidth(0.7) + fontsize(10); defaultpen(dp); pen ds = black; pen qqttff ...
Solution
Let us invert everything about O to map points to their primes. By Theorem 6 on the four collinear points M',T',S',O; we have \dfrac{M'T'}{T'O}=\dfrac{M'S'}{S'O}. Also we have the following relations:
M'T'=\dfrac{MT\times r^2}{OM\cdot OT}, T'O=\dfrac{r^2}{OT}, M'S'=\dfrac{MS\times r^2}{OM\cdot OS}, S'O=\dfrac{r^2}{OS};
And therefore we obtain MT=MS and so, M is the midpoint of TS as required.\Box

I will keep updating this post from time to time because as time goes on, the number of problems you come across also goes on.
I will add some exercises about a month later (after my secondary exams end), so, till then; Rejoice!!
References
[1] Cosmin Pohoata; Harmonic Divisions; 2007 Mathematical Reflections
[2] C. Stanley Ogilvy; (1990) Excursions in Geometry, Dover.
[3] Application 1
[4] Application 2
:)
Last Updated On: 04:30 am UTC; 11 March, 2013.

Different maddening numbers

The world is simply full of numbers of all kinds. All the clutter on numbers was made by none but man. Fibonacci and Lucas numbers; perfect numbers; Fermat numbers; Multiply-perfect numbers; abundant and super-abundant numbers, practical and impractical (!) numbers, etc. However, there is an interesting correlation between all these junk of numbers. In this entry, I try to let us get an insight into Perfect, Multiply-perfect, Abundant and Super-abundant and Practical numbers.
As usual, all of us have realized from the definition of a perfect number that what a multiply-perfect number is.
Definition and properties of the \sigma function
Let us, for the sake of clarity, define our notations first. We denote \sigma (n) as the sum of the divisors of n, and \tau (n) as the number of the divisors of n. We try to express \tau(n) in terms of the prime divisors of n. Note that, by the fundamental theorem we have that any n can be expressed in the form of (where p_i‘s are the primes arranged in order or not in order)
n=p_1^{\alpha_1}p_2^{\alpha_2}\cdots p_k^{\alpha_k};
A simple combinatorial argument leads to the obvious and well-known fact that
\tau (n)=(\alpha_1+1)(\alpha_2+1)\cdots(\alpha_n+1) (including 1 as a factor).
Now, for a formula of \sigma(n) we see that all the divisors d_i are of the form
d_i=p_1^{\beta_1}p_2^{\beta_2}\cdots p_k^{\beta_k}; and 0\leq \beta_i\leq \alpha_i. Thence the sum, can be factored into (or rather, found out to be) terms in the expansion of \prod_{i=1}^n(1+p_i+\cdots+p_i^{\alpha_i})=\prod_{i=1}^n\frac{p_i^{\alpha_i+1}-1}{p_i-1}.
Now it is evident that when \text{gcd}(m,n)=1 we have the multiplicity property of this \sigma function, ie \sigma(mn)=\sigma(m)\sigma(n). The (obvious) proof is left to the reader.
Multiply-Perfect Numbers
Before discussing the general multiply-perfect numbers, we dive into the perfect numbers for a little time. Let us try to list some of the perfect numbers. They are 6,28,496,8128,\cdots written in order. The next values can be generated with a computer, but what do we see common in these numbers?
(1) (Obviously) All are even and \sigma(n)=2n; which makes them perfect.
(2) 6=2\cdot (2^2-1); and 3=2^2-1 is a prime.
28 = 2^2\cdot (2^3-1); and 2^3-1=7 is a prime.
496 = 2^4\cdot(2^5-1); and 2^5-1=31 is a prime.
8128 = 2^6\cdot(2^7-1); and 2^7-1=127 is a prime.
Proceeding in this manner, Euler proved (again, how many theorems did he prove?) that if n is a given number with the property that 2^n-1 is a prime, that 2^{n-1}\cdot(2^{n}-1) is going to be a perfect number.
With this (extremely) brief idea of perfect numbers we are able to give a generalization. We consider a number n such that its \sigma (n)=kn where k is some natural number.
Before getting some information and problems on multiply-perfect numbers, let us try a problem.
\boxed{E1.}If n is a perfect number and n+1, n-1 are primes, find all possible values of n
Solution.
Obviously considering modulo 6 we have that 6|n\implies n=6n_1 for some n_1\in\mathbb N. If so, we consider n_1>3. In this case,
\sigma(6n_1)>1+2+3+6+n_1+2n_1+3n_1+6n_1>12n_1+12, which is way too bizarre for a perfect number 6n_1. So n_1\leq 3. So we get n=6,12,18. Indeed, 5,7 and 11,13 and 17, 18 are all primes.\Box
Let us denote a multiply perfect number of the order k by x_k whereas \sigma(x_k)=kx_k. We play around with a few multiply-perfect numbers of some orders. Just as in the previous case, we try to list some properties of a x_k number. At the first glance into a k- tuply perfect number like 120 or 672 (Verify with a calculator or the prime factorization formulae for \sigma(n)), we might try to conjecture a few things. Like, all x_k are even. We might try to construct a sequence of these numbers for different k, and conjecture that there does or does not exist such a sequence. These conjectures are needed in mathematics, but we still need a proof of those, that is why they are called conjectures. :lol:
Very few has been known about large k‘s, so let us start by noting some examples.
\boxed{E2.}Let n be a number such that 3 does not divide n yet it satisfies \sigma(n)=3n, show that O\sigma(3n)=12n, ie 3n is a perfect number of order 4.
Solution.
It is obvious from the multiplicative property of \sigma(n) that
\sigma(3n)=3\sigma(n)=12n; so that 3n is an x_4-type number.
\boxed{E3.}Every number such that \sigma(n)=5n has at least 6 different prime factors.
Solution.
This, also is obvious on using the prime decomposition of n, which is
n=p_1^{\alpha_1}\cdots p_k^{\alpha_k} gives us
\prod_{i=1}^n\frac{p_i^{\alpha_i+1}-1}{p_i^{\alpha_i}(p_i-1)}=5.
Now using \frac{p_1^{\alpha_1+1}-1}{p_1^{\alpha_1}(p_1-1)}>\frac{p_1}{p_1-1}; which is obvious on expanding, we have
\frac{p_1}{p_1-1}\cdots\frac{p_n}{p_n-1}>5.
Now with the same old technique, we consider the product upto 5 terms, which is surely going to be <5 (because the statement is modified to contradict this event by origin); therefore we are surely done.
Superabundant Numbers
A number n is known as superabundant if and only if \dfrac{\sigma(n)}{n} exceeds \dfrac{\sigma(k)}{k} for all k<n.
\boxed{E4}. The sequence of numbers t_n=\dfrac{\sigma(n)}{n} does not have any upper bound..
Solution.
If d_i are the divisors of n then we have, d_i=\dfrac n{d_{n-i}} (why?) and summing up on this sequence we get \sigma(n) as,
\sigma(n)=\sum_{d_i|n}\dfrac 1{d_i}\implies \dfrac{\sigma(n)}{n}=\sum_{d_i|n}\dfrac 1{d_i}.
Since we have to prove that this has no upper bound, let n=m! for our convenience. Then we have,
\dfrac{\sigma(n)}{n}=\sum_{d_i|n}\dfrac 1{d_i}=\sum_{d_i|m!}\dfrac1{d_i}\geq 1+\frac 12+\frac 13+\cdots+\frac 1m.
Then we have t_n\geq H_m where H is the harmonic series. As we know the harmonic series on natural numbers does not have any upper bound as n increases, and therefore t_n is unbounded from above.
Abundant Numbers
The numbers n with the property that \sigma(n)\geq 2n are abundant ones.
12, 18,20,24\cdots are abundant at our first instance, and we can also find out that all abundant numbers \leq 100 are even.
We have a property regarding the abundant numbers.
\boxed{E5} Every number n is abundant implies that mn is abundant, m\ge 2
Solution to be found by the reader.

We will discuss on some Practical, Highly Composite Numbers and Quasi-perfect, Semi-perfect numbers soon.
Hope you do not get insane with a lot of numbers, we will get some exercises on these too.

(To Be Updated Soon)